0% found this document useful (0 votes)
381 views

Dummit Foote Chapter 14 Exercises

The document contains selected exercises from Chapter 14 of Dummit & Foote, focusing on topics in field theory and automorphisms. It includes various questions and solutions related to automorphisms of fields, Galois groups, and specific field extensions. The exercises explore the properties of automorphisms, fixed fields, and isomorphisms between fields.

Uploaded by

mathcanadaschool
Copyright
© © All Rights Reserved
We take content rights seriously. If you suspect this is your content, claim it here.
Available Formats
Download as PDF, TXT or read online on Scribd
0% found this document useful (0 votes)
381 views

Dummit Foote Chapter 14 Exercises

The document contains selected exercises from Chapter 14 of Dummit & Foote, focusing on topics in field theory and automorphisms. It includes various questions and solutions related to automorphisms of fields, Galois groups, and specific field extensions. The exercises explore the properties of automorphisms, fixed fields, and isomorphisms between fields.

Uploaded by

mathcanadaschool
Copyright
© © All Rights Reserved
We take content rights seriously. If you suspect this is your content, claim it here.
Available Formats
Download as PDF, TXT or read online on Scribd
You are on page 1/ 13

Dummit & Foote Chapter 14 Selected Exercises

Igor van Loo


LinkedIn
January 31, 2024

Contents

1 Chapter 14.1 2 2.9 . . . . . . . . . . . . . . . . . . . . . 7


1.1 . . . . . . . . . . . . . . . . . . . . . 2 2.10 . . . . . . . . . . . . . . . . . . . . . 7
1.2 . . . . . . . . . . . . . . . . . . . . . 2 2.11 - Unfinished . . . . . . . . . . . . . . 7
1.3 . . . . . . . . . . . . . . . . . . . . . 2 2.12 . . . . . . . . . . . . . . . . . . . . . 7
1.4 . . . . . . . . . . . . . . . . . . . . . 3 2.13 . . . . . . . . . . . . . . . . . . . . . 8
1.5 . . . . . . . . . . . . . . . . . . . . . 3 2.14 . . . . . . . . . . . . . . . . . . . . . 8
1.6 . . . . . . . . . . . . . . . . . . . . . 3 2.15 . . . . . . . . . . . . . . . . . . . . . 8
1.7 . . . . . . . . . . . . . . . . . . . . . 4 2.16 . . . . . . . . . . . . . . . . . . . . . 9
2.17 . . . . . . . . . . . . . . . . . . . . . 10
2 Chapter 14.2 4
2.1 . . . . . . . . . . . . . . . . . . . . . 4 3 Chapter 14.3 11
2.2 . . . . . . . . . . . . . . . . . . . . . 5 3.1 . . . . . . . . . . . . . . . . . . . . . 11
2.3 . . . . . . . . . . . . . . . . . . . . . 5 3.2 . . . . . . . . . . . . . . . . . . . . . 11
2.4 . . . . . . . . . . . . . . . . . . . . . 5 3.3 . . . . . . . . . . . . . . . . . . . . . 12
2.5 . . . . . . . . . . . . . . . . . . . . . 6 3.4 . . . . . . . . . . . . . . . . . . . . . 12
2.6 . . . . . . . . . . . . . . . . . . . . . 6 3.5 . . . . . . . . . . . . . . . . . . . . . 12
2.7 - Unfinished . . . . . . . . . . . . . . 6 3.6 - Unfinished . . . . . . . . . . . . . . 13
2.8 - Unfinished . . . . . . . . . . . . . . 6 3.7 . . . . . . . . . . . . . . . . . . . . . 13

1
1 Chapter 14.1

Question 1.1

1. Show that if the field 𝐾 is generated over 𝐹 by the elements 𝑎1 , ⋯ , 𝑎𝑛 then an automorphism 𝜎 of
𝐾 fixing 𝐹 is uniquely determined by 𝜎(𝑎1 ), ⋯ , 𝜎(𝑎𝑛 ). In particular, show that an automorphism
fixes 𝐾 if and only if it fixes a set of generators for 𝐾
2. Let 𝐺 ≤ Gal(𝐾 /𝐹 ) be a subgroup of the Galois group of the extension 𝐾 /𝐹 and suppose 𝜎1 , ⋯ , 𝜎𝑘
are generators for 𝐺. Show that the subfield 𝐸/𝐹 is fixed by 𝐺 if and only if it is fixed by the
generators 𝜎1 , ⋯ , 𝜎𝑘

Solution.
𝑗 𝑗
(a) Let 𝑥 ∈ 𝐾 = 𝐹 (𝛼1 , ⋯ , 𝛼𝑛 ), then we have that 𝑥 = 𝑎1 𝜃1 + ⋯ + 𝑎𝑚 𝜃𝑚 where 𝜃𝑖 = 𝛼1𝑖,1 ⋯ 𝛼𝑛𝑖,𝑛 . (This is basically
saying that each element in 𝐾 is expressed √ √as a linear combination
√ √ of
√ all possible products of the 𝛼𝑖 ’s,
which is obviously true, for example, ℚ( 2, 3) = {𝑎 + 𝑏 2 + 𝑐 3 + 𝑑 6 ∣ 𝑎, 𝑏, 𝑐, 𝑑 ∈ ℚ})
Then, we have that
𝜎(𝑥) = 𝜎(𝑎1 𝜃1 + ⋯ + 𝑎𝑚 𝜃𝑚 ) = 𝑎1 𝜎(𝜃1 ) + ⋯ + 𝑎𝑚 𝜎(𝜃𝑚 )
since 𝜎 is a homomorphism that fixes 𝐹 . Furthermore,
𝑗 𝑗
𝜎(𝜃𝑖 ) = 𝜎(𝛼1𝑖,1 ⋯ 𝛼𝑛𝑗𝑖,𝑛 ) = 𝜎(𝛼1𝑖,1 ) ⋯ 𝜎(𝛼𝑛𝑗𝑖,𝑛 )

and therefore 𝜎(𝑥) is determined by 𝜎(𝑎1 ), ⋯ , 𝜎(𝑎𝑛 ).


In particular, 𝜎(𝑥) = 𝑥 ∈ 𝐾 ⟺ 𝜎(𝛼𝑖 ) = 𝛼𝑖 for 1 ≤ 𝑖 ≤ 𝑛
(b) Let 𝐺 = ⟨𝜎1 , … , 𝜎𝑘 ⟩. That is, any element 𝜎 ∈ 𝐺 can be written in the form
𝑚
𝑛
𝜎 = ∏ 𝛾𝑗 𝑗
𝑗=1

where each 𝛾𝑗 ∈ {𝜎𝑖 ∣ 1 ≤ 𝑖 ≤ 𝑘} (note that 𝛾𝑗 are not necessarily distinct, in fact there are likely to be
repeats), and 𝑛𝑗 ∈ ℤ.
By assumption, each 𝜎𝑖 ↾𝐸 = 1, that is 𝜎𝑖𝑛𝑖 (𝑥) = 𝑥 for any 𝑥 ∈ 𝐸 and 𝑛𝑖 ∈ ℤ. It follows immediately that
𝑚
𝑛
𝜎(𝑥) = (∏ 𝛾𝑗 𝑗 )(𝑥) = 𝑥.
𝑗=1

Question 1.2

Let 𝜏 be the map 𝜏 ∶ ℂ → ℂ defined by 𝜏(𝑎 + 𝑏𝑖) = 𝑎 − 𝑏𝑖. Prove that 𝜏 is an automorphism of ℂ

Solution. It is easily shown that 𝜏 is a homomorphism and that it is bijective and hence 𝜏 is an isomorphism □

Question 1.3

Determine the fixed field of complex conjugation on ℂ

Solution. The fixed field of complex conjugation is 𝐹 = {𝑎 + 𝑏𝑖 ∈ ℂ ∣ 𝜏(𝑎 + 𝑏𝑖) = 𝑎 + 𝑏𝑖}, therefore we need
𝜏(𝑎 + 𝑏𝑖) = 𝑎 − 𝑏𝑖 = 𝑎 + 𝑏𝑖 ⟹ 2𝑏𝑖 = 0 ⟹ 𝑏 = 0 therefore 𝑎 + 𝑏𝑖 ∈ 𝐹 ⟺ 𝑏 = 0. In this case we have
𝑎 + 𝑏𝑖 = 𝑎 ∈ ℝ and therefore 𝐹 = ℝ □

2
Question 1.4

√ √
Prove that ℚ( 2) and ℚ( 3) are not isomorphic

Solution. An important note is that these 2 fields are isomorphic as vector


√ spaces √ over ℚ, however, they
are not field isomorphic. We√have previously
√ shown in Chapter 13 that√ 22 ∉ ℚ( 3) and therefore if there
was an isomorphism 𝜑 ∶ ℚ( 2) → ℚ( 3) then we can notice that 𝜑( 2) √ = √ = 2 because
𝜑(2) √ ℚ is fixed
(Alternatively you can use the simpler fact that 𝜎(1) = 1) which implies 𝜑( 2) = ± 2 ∉ ℚ( 3) and therefore
this isomorphism cannot exist. □

Question 1.5

√ √
Determine the automorphisms of the extension ℚ( 4 2)/ℚ( 2) explicitly

√4 √ √ √
√4 First we note that [ℚ( 2) ∶ ℚ( 2)] = 2 and we have minimal polynomial 𝑥 − 2 ∈ ℚ( 2) with
Solution. 2

roots ± 2 and therefore we can only have 2 automorphisms


√ √
1 ∶ 4 2 ↦ 4 2 (𝐼 𝑑𝑒𝑛𝑡𝑖𝑡𝑦)
√ √
𝜎 ∶ 42 ↦ − 42

Question 1.6

Let 𝑘 be a field
(a) Show that the mapping 𝜑 ∶ 𝑘[𝑡] → 𝑘[𝑡] defined by 𝜑(𝑓 (𝑡)) = 𝑓 (𝑎𝑡 + 𝑏) for fixed 𝑎, 𝑏 ∈ 𝑘, 𝑎 ≠ 0 is an
automorphism of 𝑘[𝑡] which is the identity on 𝑘

(b) Conversely, let 𝜑 be an automorphism of 𝑘[𝑡] which is the identity on 𝑘. Prove that there exist
𝑎, 𝑏 ∈ 𝑘 with 𝑎 ≠ 0 such 𝜑(𝑓 (𝑡)) = 𝑓 (𝑎𝑡 + 𝑏) as in (𝑎)

Solution.
(a) Let 𝑓 (𝑡), 𝑔(𝑡) ∈ 𝑘[𝑡], then we show that 𝜑 is an isomorphism.
𝜑((𝑓 + 𝑔)(𝑡)) = (𝑓 + 𝑔)(𝑎𝑡 + 𝑏) = 𝑓 (𝑎𝑡 + 𝑏) + 𝑔(𝑎𝑡 + 𝑏) = 𝜑(𝑓 (𝑡)) + 𝜑(𝑔(𝑡))
and
𝜑((𝑓 𝑔)(𝑡)) = (𝑓 𝑔)(𝑎𝑡 + 𝑏) = 𝑓 (𝑎𝑡 + 𝑏)𝑔(𝑎𝑡 + 𝑏) = 𝜑(𝑓 (𝑡))𝜑(𝑔(𝑡))
Therefore 𝜑 is a homomorphism. Now, suppose 𝜑(𝑓 (𝑡)) = 𝜑(𝑔(𝑡)). Then 𝑓 (𝑎𝑡 + 𝑏) = 𝑔(𝑎𝑡 + 𝑏) and because
𝑘[𝑎𝑡 + 𝑏] = 𝑘[𝑡] we have that 𝑓 (𝑡) = 𝑔(𝑡). Lastly let 𝑔(𝑡) ∈ 𝑘[𝑡] then take 𝑓 (𝑡) = 𝑔( 𝑎𝑡 − 𝑎𝑏 ) ∈ 𝑘[𝑡] and we
have 𝜑(𝑓 (𝑡)) = 𝜑(𝑓 (𝑎𝑡 + 𝑏)) = 𝑔(𝑎( 𝑎𝑡 − 𝑎𝑏 ) + 𝑏) = 𝑔(𝑡) and therefore 𝜑 is bijective, finally we conclude 𝜑 is
an isomorphism.
Lastly, if 𝑓 (𝑡) = 𝑐 ∈ 𝑘 ⊂ 𝑘[𝑡] then 𝜑(𝑓 (𝑡)) = 𝑓 (𝑎𝑡 + 𝑏) = 𝑐 and therefore 𝜑 is the identity on 𝑘
(b) Suppose 𝜑(𝑓 (𝑡)) = ℎ(𝑡)𝑓 (𝑡) + 𝑔(𝑡) where 𝑔(𝑡), ℎ(𝑡) ∈ 𝑘[𝑡] then because 𝜑 is identity on 𝑘 we would have
𝜑(𝑐) = ℎ(𝑡)𝑐 + 𝑔(𝑡) = 𝑐 ⟹ 𝑔(𝑡) = 0, ℎ(𝑡) = 1 therefore we must have that 𝜑(𝑓 (𝑡)) = 𝑓 (𝑔(𝑡)) for some
𝑔(𝑡) ∈ 𝑘[𝑡].
We want 𝑔(𝑡) = 𝑎𝑡 + 𝑏 therefore we must show that if deg(𝑔(𝑡)) ≥ 2 there is a contradiction.
Suppose deg(𝑔(𝑡)) ≥ 2 this implies that the deg(𝑓 (𝑔(𝑡)) ≥ 2 and therefore this map is not surjective,
therefore we conclude deg(𝑔(𝑡)) ≤ 1.
If deg(𝑔(𝑡)) = 0 then 𝑔(𝑡) = 𝑏 ∈ 𝑘 and this map is not injective.
Finally, we conclude that deg(𝑔(𝑡)) = 1 and therefore 𝑔(𝑡) = 𝑎𝑡 + 𝑏 where 𝑎, 𝑏 ∈ 𝑘 and 𝜑(𝑓 (𝑡)) = 𝑓 (𝑔(𝑡)) =
𝑓 (𝑎𝑡 + 𝑏)

3

Question 1.7

This exercise determines Aut(ℝ/ℚ)


(a) Prove that any 𝜎 ∈ Aut(ℝ/ℚ) takes squares to squares and takes positive reals to positive reals.
Conclude that 𝑎 < 𝑏 implies 𝜎(𝑎) < 𝜎(𝑏) for every 𝑎, 𝑏 ∈ ℝ

(b) Prove that any − 𝑚1 < 𝑎 − 𝑏 < 𝑚1 implies − 𝑚1 < 𝜎(𝑎) − 𝜎(𝑏) < 1
𝑚 for every positive integer 𝑚.
Conclude that 𝜎 is a continuous map on ℝ
(c) Prove that any continuous map on ℝ which is the identity on ℚ is the identity map, hence
Aut(ℝ/ℚ) = 1

Solution.

(a) Let 𝑎 ∈ ℝ be a square. That is, ∃𝑏 ∈ ℝ s.t. 𝑏2 = 𝑎. Then 𝜎(𝑎) = 𝜎(𝑏2 ) = (𝜎(𝑏))2 . That is, 𝜎 takes squares to
squares. Since the only squares in ℝ are the non-negative reals, but 𝜎(𝑎) = 0 ⟹ 𝑎 = 0, so it must be
that 𝜎 takes positive reals to positive reals.
Suppose now that 𝑏 − 𝑎 > 0, then 𝜎(𝑏 − 𝑎) > 0, giving that 𝜎(𝑏) − 𝜎(𝑎) > 0.
(b) Since ∀𝜎 ∈ Aut(ℝ/ℚ), 𝜎 fixes ℚ, then
1 1
− <𝑎−𝑏<
𝑚 𝑚
1 1
𝜎 − < 𝜎(𝑎 − 𝑏) < 𝜎 , 𝜎 preserves order by part (a)
( 𝑚) (𝑚)

− < 𝜎(𝑎) − 𝜎(𝑏) < , 𝜎 ↾ℚ = 1


1 1
𝑚 𝑚

Now we prove continuity. Let 𝜀 > 0 and take |𝑎 − 𝑏| < 𝛿 = 1


𝑚 < 𝜀 then we have that |𝜎(𝑎) − 𝜎(𝑏)| < 1
𝑚 <𝜀
(c) (Method 1) Let 𝑥 ∈ ℝ, suppose 𝑥 < 𝜎(𝑥) then ∃𝑞 ∈ ℚ such that 𝑥 < 𝑞 < 𝜎(𝑥) and then using 𝑥 < 𝑞 we
have from part (a) 𝜎(𝑥) < 𝜎(𝑞) = 𝑞 and therefore 𝑥 = 𝜎(𝑥) which is a contradiction. Similarly
if 𝑥 > 𝜎(𝑥) we get a contradiction, therefore we conclude 𝑥 = 𝜎(𝑥), ∀𝑥 ∈ ℝ
(Method 2) Let 𝑥 ∈ ℝ, 𝜀 > 0. Since 𝜎 is continuous we know ∃𝛿1 > |𝑥 − 𝑦| such that |𝜎(𝑥) − 𝜎(𝑦)| < 2𝜀 .
Take 𝑎 ∈ ℚ such that |𝑎 − 𝑥| < min{ 2𝜀 , 𝛿1 } then we have that
𝜀 𝜀
|𝜎(𝑥) − 𝑥| = |𝜎(𝑥) − 𝑎 + 𝑎 − 𝑥| = |𝜎(𝑥) − 𝑎| + |𝑎 − 𝑥| = |𝜎(𝑥) − 𝜎(𝑎)| + |𝑎 − 𝑥| < + =𝜀
2 2
which shows that |𝜎(𝑥) − 𝑥| < 𝜀, ∀𝑥 ∈ ℝ

2 Chapter 14.2

Question 2.1

√ √
Determine the minimal polynomial over ℚ for the element 2+ 5

√ √ √ √
Solution.
√ ℚ( √ 2 + 5) ⊂ ℚ( 2, 5) which is Galois over ℚ and therefore the roots
√ of√the minimal
√ polynomial

are
√ ± 2
√ ± 5 which
√ are
√ all distinct. Hence the minimal polynomial is (𝑥 − ( 2 + 5)(𝑥 + ( 2 + 5))(𝑥 −
( 2 − 5))(𝑥 + ( 2 − 5)) = 𝑥 4 − 14𝑥 2 + 9 □

4
Question 2.2

√ √
Determine the minimal polynomial over ℚ for the element 1 + 3 2 + 3 4

√3 √3 √ √ 2𝜋𝑖
Solution. We have shown in √3 chapter 13 that ℚ(1 √3 +√3 2 +
√3 4) ⊂ ℚ( 3 2) ⊂ ℚ( 3 2, 𝜁 ) where 𝜁 = 𝑒 3 which√is a
Galois extension, therefore 2 must be sent to 2, 2𝜁 , 2𝜁 2 and notice that we only care about where 3 2 is
√ 2 √ √ 3
sent as 3 2 = 3 4, 3 2 = 1.
Knowing this we know that the 3 roots of our minimal polynomial are
√ √
𝑟1 = 1 + 3 2 + 3 4
√ √
𝑟2 = 1 + 3 2𝜁 + 3 4𝜁 2
√ √
𝑟3 = 1 + 3 2𝜁 2 + 3 4𝜁
√ √
Painfully
√3 √expanding (𝑥 −𝑟1√)(𝑥 −𝑟√2 )(𝑥 −𝑟3 ) gives you 𝑥 3 −3𝑥 2 −3𝑥 −1. Alternatively (𝑟1 −1)3 = ( 3 2+ 3 4)3 =
2 + 3 16 + 3 3 32 + 4 = 6 + 6( 3 2 + 3 4) = 6 + 6(𝑟1 − 1) = 6𝑟1 □

Question 2.3

Determine the Galois group of 𝑓 = (𝑥 2 − 2)(𝑥 2 − 3)(𝑥 2 − 5). Determine all subfields of the splitting
field of 𝑓

√ √ √ √ √
Solution. The splitting field of 𝑓 is clearly 𝐾 = ℚ( 2, 3, 5) and any automorphism of 𝐾 will map 𝑎 → ± 𝑎
where 𝑎 ∈ {2, 3, 5} and therefore there are 8 total automorphisms. Now we must show that there are no more
than 8, this is done by noting that | Aut(𝐾 /ℚ)| ≤ [𝐾 ∶ ℚ] = 8, furthermore we can conclude that this
extensions is Galois. The subfields are

ℚ( 𝑎) where 𝑎 ∈ {2, 3, 5, 6, 10, 15, 30}
√ √ √ √ √ √ √ √ √ √ √ √ √ √
ℚ( 2, 3), ℚ( 2, 5), ℚ( 3, 5), ℚ( 2, 15), ℚ( 3, 10), ℚ( 5, 6), ℚ( 10, 15)

Question 2.4

Let 𝑝 be a prime. Determine the elements of the Galois group of 𝑥 𝑝 − 2


Solution. The splitting field of 𝑥 𝑝 − 2 is 𝐾 = ℚ( 𝑝 2, 𝜁 ) where 𝜁 is the p-th root of unity.
√ √
1. Consider 𝐺1 = Gal(𝐾 /ℚ(𝜁 )) and 𝜏( 𝑝 2) = 𝑝 2𝜁 and it fixes 𝜁 . The order of 𝜏 is 𝑝 and therefore 𝐺1 ≅
⟨𝜏 ⟩ ≅ 𝐶𝑝
√ √
2. Consider 𝐺2 = Gal(𝐾 /ℚ( 𝑝 2)) and 𝜎(𝜁 ) = 𝜁 𝑎 and it fixes 𝑝 2. The order of 𝜎 is 𝑝 − 1 because 𝑎𝑝−1 ≡ 1
(mod 𝑝) and therefore 𝐺2 ≅ ⟨𝜎 ⟩ ≅ 𝐶𝑝−1
Furthermore, we know the following:
√ √
1. [𝐾 ∶ ℚ] = [𝐾 ∶ ℚ( 𝑝 2)][ℚ( 𝑝 2) ∶ ℚ] = (𝑝 − 1)𝑝 is a galois extension and hence |𝐺| = | Gal(𝐾 /ℚ)| =
𝑝(𝑝 − 1)
2. |⟨𝜏 ⟩||⟨𝜎 ⟩| = 𝑝(𝑝 − 1)

3. |⟨𝜏 ⟩ ⋂ ⟨𝜎 ⟩| = 1
|⟨𝜏 ⟩||⟨𝜎 ⟩|
Therefore, using point 2 and 3 and the following |⟨𝜏 ⟩⟨𝜎 ⟩| = we have that 𝐺 = ⟨𝜏 ⟩⟨𝜎 ⟩. Futher-
|⟨𝜏 ⟩ ⋂ ⟨𝜎 ⟩|
more we can notice that 𝐾 𝐺1 /ℚ = ℚ(𝜁 )/ℚ is a galois extension because [ℚ(𝜁 ) ∶ ℚ] = 𝑝 − 1 = | Aut(ℚ(𝜁 )/ℚ)|
and therefore 𝐺1 ⊲ 𝐺 and therefore we have 𝐺 ≅ 𝐶𝑝 ⋊ 𝐶𝑝−1 □

5
Question 2.5

Prove that the Galois group of 𝑥 𝑝 − 2 for 𝑝 a prime is isomorphic to the group of matrices

𝑎 𝑏
(0 1)

where 𝑎, 𝑏 ∈ 𝔽𝑝 , 𝑎 ≠ 0

√ √
Solution. Let 𝐺 = Gal(ℚ( 𝑝 2, 𝜁 )/ℚ). Now notice
√ that√any element 𝜑 ∈ 𝐺 is determined by 𝜑(𝜁 ) and 𝜑( 𝑝 2),
where 𝜑(𝜁 ) = 𝜁 𝑎 for some 1 ≤ 𝑖 ≤ 𝑝 − 1 and 𝜑( 𝑝 2) = 𝑝 2𝜁 𝑏 for some 0 ≤ 𝑏 ≤ 𝑝 − 1 then we define the map

𝑎 𝑏
𝛼∶𝐺→{ where 𝑎, 𝑏 ∈ 𝔽𝑝 , 𝑎 ≠ 0}
(0 1)

𝑎 𝑏
𝛼(𝜑) =
(0 1)
This is a homomorphism and bijective, hence an isomorphism □

Question 2.6

√ √ √
Let 𝐾 = ℚ( 8 2, 𝑖) and let 𝐹1 = ℚ(𝑖), 𝐹2 = ℚ( 2), 𝐹3 = ℚ(− 2). Prove that Gal(𝐾 /𝐹1 ) ≅ ℤ8 , Gal(𝐾 /𝐹2 ) ≅
𝐷8 , Gal(𝐾 /𝐹3 ) ≅ 𝑄8

Solution. We follow the discussion from Chapter 14.2 where we found that
⎧√8 √ ⎧√8 √

⎪ 2 → 𝜁 82 ⎪
⎪ 2 → 82
⎪ ⎪
Gal(𝐾 /ℚ) = ⟨𝜎, 𝜏 ∶ 𝜎 8 = 𝜏 2 = 1, 𝜎𝜏 = 𝜏𝜎 3 ⟩ where 𝜎 = ⎨𝑖 → 𝑖 and 𝜏 = ⎨𝑖 → −𝑖

⎪ 5

⎪ 7
⎩𝜁 → 𝜁
⎪ ⎩𝜁 → 𝜁

1. Clearly 𝜎 fixes 𝑖 therefore Gal(𝐾 /𝐹1 ) = ⟨𝜎 ⟩ ≅ ℤ8


√ √ √ √
2. 𝜏 fixes 2 already, now we need 𝜎 𝑛 ( 2) = 𝜎 𝑛 ( 8 2)4 = 2𝜁 4𝑛 , we need 𝜁 4𝑛 = 1 ⟹ 𝑛 = 2, 4, 6,
therefore Gal(𝐾 /𝐹2 ) = {1, 𝜎 2 , 𝜎 4 , 𝜎 6 , 𝜏, 𝜏𝜎 2 , 𝜏𝜎 4 , 𝜏𝜎 6 } = ⟨𝜎 2 , 𝜏 ⟩ where (𝜎 2 )4 = 𝜏 2 = 1 and 𝜎 2 𝜏 = 𝜏𝜎 6
which describes 𝐷8 and therefore Gal(𝐾 /𝐹2 ) ≅ 𝐷8
√ √ √ √ 4 √
3. Note that −2 = 2𝑖( 8 2)4 𝑖, clearly √ 𝜏 will not√fix this. We try √ 𝜎 𝑛 (( 8 2)√𝑖) = 𝜁 4𝑛 2𝑖 therefore 𝑛 = 2, 4, 6
from part 2. Next we try 𝜏𝜎 𝑛 (( 8 2)4 𝑖) = 𝜏(𝜁 4 𝑛 2𝑖) = −𝜁 28𝑛 2𝑖 = −𝜁 4𝑛 2𝑖, we need −𝜁 4 = 1 ⟹ 𝑛 =
1, 3, 5, 7 therefore Gal(𝐾 /𝐹3 ) = {1, 𝜎 2 , 𝜎 4 , 𝜎 6 , 𝜏𝜎, 𝜏𝜎 3 , 𝜏𝜎 5 , 𝜏𝜎 7 } = ⟨𝜎 2 , 𝜏𝜎 3 ⟩ with the relations (𝜎 2 )4 =
1, (𝜎 2 )2 = 𝜎 4 = (𝜏𝜎 3 )2 , 𝜏𝜎 4 = (𝜎 2 )−1 𝜏𝜎 3 which describes 𝑄8 and therefore Gal(𝐾 /𝐹3 ) ≅ 𝑄8

Question 2.7 - Unfinished

Determine all the subfields of the splitting field of 𝑥 8 − 2 which are Galois over ℚ

Question 2.8 - Unfinished

Suppose 𝐾 is a Galois extension of 𝐹 of degree 𝑝𝑛 for some prime 𝑝 and some 𝑛 ≥ 1. Show there are
Galois extensions of 𝐹 contained in 𝐾 of degrees 𝑝 and 𝑝𝑛−1

6
Question 2.9

Give an example of fields 𝐹1 , 𝐹2 , 𝐹3 with ℚ ⊂ 𝐹1 ⊂ 𝐹2 ⊂ 𝐹3 , [𝐹3 ∶ ℚ] = 8 and each field if Galois over all
of its subfields with the exception that 𝐹2 is not Galois over ℚ

√ √ √
Solution. Take 𝐹3 = ℚ( 4 2, 𝑖), 𝐹2 = ℚ( 4 2), 𝐹1 = ℚ( 2). Then we have that 𝐹3 is Galois over 𝐹2 , 𝐹1 , ℚ, 𝐹2 is
Galois over 𝐹1 but not Galois over ℚ and 𝐹1 is Galois over ℚ □

Question 2.10

Determine the Galois group of the splitting field over ℚ of 𝑥 8 − 3

√ √ √
Solution. The splitting field of the polynomial is 𝐾 = ℚ( 8 3, 𝜁 ) = ℚ( 8 3, 2, 𝑖) where
√8 𝜁 √is an 8-th
√8 root
√ of
unity.
√ This√extension is of degree 32 because of the√ following,
√ [𝐾 ∶ ℚ] = [𝐾 ∶ ℚ( 3, 2)][ℚ( 3, 2) ∶
ℚ( 2)][ℚ( 2) ∶ ℚ] = 32 because we can show that 8 3 ∉ ℚ( 2).
Any automorphism of Aut(𝐾 /ℚ) is of the form
√8 √ √ √
3 ↦ 8 3𝜁 𝑖 , 1 ≤ 𝑖 ≤ 7, 2 ↦ ± 2, 𝑖 ↦ ±𝑖

Alternatively, consider the generators


√ √
1. 𝜎 ∶ 8 3 ↦ 8 3𝜁 .

2. 𝜏𝑖 ∶ 𝜁 ↦ 𝜁 𝑖 , for 𝑖 ∈ {3, 5, 7}
and work out the relations. Namely, all automorphisms can be written in the form 𝜎 𝑎 , 𝜎 𝑎 𝜏3 , 𝜎 𝑎 𝜏5 , 𝜎 𝑎 𝜏7 for
0 ≤ 𝑎 ≤ 7, giving exactly 32 automorphisms as desired. □

Question 2.11 - Unfinished

Suppose 𝑓 (𝑥) ∈ ℤ[𝑥] is an irreducible quartic whose splitting field has Galois group 𝑆4 over ℚ (there
are many such quartics, cf. Section 6). Let 𝜃 be a root of 𝑓 (𝑥) and set 𝐾 = ℚ(𝜃). Prove that 𝐾 is an
extension of ℚ of degree 4 which has no proper subfields. Are there any Galois extensions of ℚ of
degree 4 with no proper subfields?

Question 2.12

Determine the Galois group of the splitting field over ℚ of 𝑥 4 − 14𝑥 2 + 9.

√ √
√ √ Note: From Question 2.1
Solution. √ we√ can already see that the splitting field of the polynomial is ℚ( 2+ 5) =
ℚ( 2, 5) and therefore Gal(ℚ( 2 + 5)/ℚ) ≅ 𝐾4 , now we can just confirm the answer.
Solving for 𝑥 2 using the quadratic formula we see that

2 14 ± 142 − 4(1)(9) √ √ √
𝑥 = = 7 ± 2 10 = ( 2 ± 5)2
2
√ √
Then,√we have
√ that the√ roots
√ of the polynomial are ± 2± 5 and therefore the splitting field of the polynomial
is ℚ( 2 + 5) = ℚ( 2, 5) which has 4 automorphisms.
Finally, we conclude
{√ √ {√ √
√ √ 2→ 2 2→− 2
Gal(ℚ( 2 + 5)/ℚ) = {1, 𝜎, 𝜏, 𝜎𝜏 = 𝜏𝜎} ≅ 𝐾4 where 𝜎 = √ √ and 𝜏 = √ √
5→− 5 5→ 5

7
Question 2.13

Prove that if the Galois group of the splitting field of a cubic over ℚ is the cyclic group of order 3 then
all the roots of the cubic are real.

Solution. Suppose the 3 roots are not all real, then we must have one real root 𝑟1 and 2 complex roots 𝑧, 𝑧 in
which case the splitting field would be ℚ(𝑟1 , 𝑧) and we have an automorphism of Gal(ℚ(𝑟1 , 𝑧)/ℚ) which would
fix 𝑟1 and send 𝑧 ↦ 𝑧 and therefore 2 would divide | Gal(ℚ(𝑟1 , 𝑧)/ℚ)| and hence Gal(ℚ(𝑟1 , 𝑧)/ℚ) ≇ ℤ3 □

Question 2.14

√ √
Show that ℚ( 2 + 2) is a cyclic quartic field, i.e, is a Galois extension of degree 4 with cyclic Galois
group

√ √ √ √
Solution.
√ Let 𝐾 = ℚ( √ 2 + 2). We find a polynomial with root 𝑥 = 2 + 2 using the following 𝑥 2 =
2 + 2 ⟹ 𝑥 2 − 2 = 2 ⟹ 𝑥 4 − 4𝑥 2 + 4 = 2 ⟹ 𝑥 4 − 4𝑥 2 + 2 which √ is a degree 4 polynomial and is

irreducible
√ by Eisenstein criterion, therefore
√ it is the

minimum polynomial of 2 + 2 over ℚ. The 4 roots are
√ √
± 2 ± 2 and we can notice that 2 − 2 = √ 2√ ∈ 𝐾 , so all our roots are contained in 𝐾 which makes 𝐾
2+ 2
the splitting field of a separable polynomial (as the roots are distinct) and therefore
√ a Galois
√ Extension of ℚ,
√ √
hence | Aut(𝐾 /ℚ)| = [𝐾 ∶ ℚ] = 4. Furthermore, if 𝜎 ∈ Gal(𝐾 /ℚ) such that 𝜎( 2 + 2) = 2 − 2 we have
that
√ √ √ √ √
√ √ √ √ 2 𝜎( 2) 𝜎(( 2 + 2)2 − 2) − 2 √ √
2
𝜎 ( 2 + 2) = 𝜎( 2 − 2) = 𝜎( √ √ )= √ √ = √ √ =√ √ =− 2− 2
2+ 2 𝜎( 2 + 2) 2− 2 2+ 2
Therefore ord(𝜎) > 2 and it must divide 4, which implies that ord(𝜎) = 4 and therefore Gal(𝐾 /ℚ) ≅ ℤ4 □

Question 2.15

(Biquadratic extensions) Let 𝐹 be a field of characteristic ≠ 2


√ √
(a) If 𝐾 = 𝐹 ( 𝐷1 , 𝐷2 ) where 𝐷1 , 𝐷2 ∈ 𝐹 have the property than none of 𝐷1 , 𝐷2 , 𝐷1 𝐷2 is a square in
𝐹 , prove that 𝐾 /𝐹 is a Galois extension with Gal(𝐾 /𝐹 ) isomorphic to the Klein 4 group
√ √
(b) Conversly, suppose 𝐾 /𝐹 is a Galois extension with Gal(𝐾 /𝐹 ) ≅ 𝐾4 . Prove that 𝐾 = 𝐹 ( 𝐷1 , 𝐷2 )
where 𝐷1 , 𝐷2 ∈ 𝐹 have the property that none of 𝐷1 , 𝐷2 , 𝐷1 𝐷2 is square in 𝐹

Solution.
√ √
(a) If 𝐷1 , 𝐷2 are not square in 𝐹 this implies that [𝐹 ( 𝐷1 ) ∶ 𝐹 ] = [𝐹 ( 𝐷2 ) ∶ 𝐹 ] = 2 and therefore
√ √ √ √
[𝐾 ∶ 𝐹 ] = [𝐾 ∶ 𝐹 ( 𝐷1 )][𝐹 ( 𝐷1 ) ∶ 𝐹 ] ≤ [𝐹 ( 𝐷1 ) ∶ 𝐹 ][𝐹 ( 𝐷2 ) ∶ 𝐹 ] = 4
√ √ √ √
We then √ have that√[𝐾 ∶ 𝐹 ( 𝐷1 )] ≤ √ 2. To show√that [𝐾 ∶ 𝐹 ( 𝐷1 )] = 2 we show that 𝐷2√∉ 𝐹 ( 𝐷1 ).
Suppose 𝐷2 ∈ 𝐹 ( 𝐷1 ) then we have 𝐷2 = 𝑎+𝑏 𝐷1 where 𝑎, 𝑏 ∈ 𝐹 , this implies 𝐷2 = 𝑎2 +2𝑎𝑏 𝐷1 +𝑏2 𝐷1 ,
because 𝐷2 is not square in 𝐹 we must have 𝑎 = 0 or 𝑏 = 0. If 𝑏 = 0 then 𝐷2 = 𝑎2 which means 𝐷2 is a
square, a contradiction. If 𝑎 = 0 then√ 𝐷2 = √ 𝑏2 𝐷1 ⟹ 𝐷1 𝐷2 = 𝑏2 𝐷12 which means 𝐷1 𝐷2 is a square, a
contradiction. Hence we conclude 𝐷2 ∉ 𝐹 ( 𝐷1 ) and therefore [𝐾 ∶ 𝐹 ] = 4. Furthermore, it is easy to
see that we have 4 automorphisms of 𝐾 fixing 𝐹
{√ √ {√ √ {√ √
𝐷1 ↦ − 𝐷1 𝐷1 ↦ 𝐷1 𝐷1 ↦ − 𝐷1
𝐼𝑑 𝜎 = √ √ 𝜏= √ √ 𝜎𝜏 = 𝜏𝜎 = √ √
𝐷2 ↦ 𝐷2 𝐷2 ↦ − 𝐷2 𝐷2 ↦ − 𝐷2

and hence we conclude that 𝐾 /𝐹 is a Galois extension with Gal(𝐾 /𝐹 ) ≅ 𝐾4

8
(b) Given that Gal(𝐾 /𝐹 ) ≅ 𝐾4 and 𝐾4 has 3 non-trivial subgroups or order 2; ⟨1, 𝜎⟩, ⟨1, 𝜏⟩, ⟨1, 𝜎𝜏⟩ there will be
correspondingly
√ √3 subfields 𝐸1 , 𝐸2 , 𝐸3 of 𝐾 containing 𝐹 where they are degree 2 extensions of 𝐹 . Let 𝐸1 =
𝐹 ( 𝐷1 ), 𝐸2 = 𝐹 ( 𝐷2 ) where 𝐷1 , 𝐷2 are not square √ in 𝐹 as needed, then the fact that 𝐸1 ≠ 𝐸2 ⟹ 𝐷1 𝐷2 is
not square in 𝐹 from part (a), therefore 𝐸3 = 𝐹 ( 𝐷1 𝐷2 ) is a degree 2 extension √ of 𝐹√. Finally, we have that
𝐸1 𝐸2 is a degree 4 extension over 𝐹 and 𝐸1 , 𝐸2 , 𝐸3 ⊂ 𝐸1 𝐸2 ⟹ 𝐾 = 𝐸1 𝐸2 = 𝐹 ( 𝐷1 , 𝐷2 )

Question 2.16

(a) Prove that 𝑥 4 − 2𝑥 2 − 2 is irreducible over ℚ

(b) Show that the roots of this quartic are


√ √ √ √
𝛼1 = 1 + 3 𝛼3 = − 1 + 3
√ √ √ √
𝛼2 = 1 − 3 𝛼4 = − 1 − 3

(c) Let 𝐾1 = ℚ(𝛼1 ) and 𝐾2 = ℚ(𝛼2 ). Show that 𝐾1 ≠ 𝐾2 and 𝐾1 ∩ 𝐾2 = ℚ( 3) = 𝐹 .
(d) Prove that 𝐾1 , 𝐾2 and 𝐾1 𝐾2 are Galois over 𝐹 with Gal(𝐾1 𝐾2 /𝐹 ) the Klein 4-group. Write out the
elements of Gal(𝐾1 𝐾2 /𝐹 ) explicitly. Determine all the subgroups of the Galois group and give their
corresponding fixed subfields of 𝐾1 𝐾2 containing 𝐹 .
(e) Prove that the splitting field of 𝑥 4 − 2𝑥 2 − 2 over ℚ is of degree 8 with dihedral Galois group

Solution.

(a) Using Eisenstein with 𝑝 = 2 shows that 𝑥 4 − 2𝑥 2 − 2 is irreducible over ℚ


√ √ √ √ √ √ √ √ √ √
(b) (𝑥 − 1 + 3)(𝑥 + 1 + 3)(𝑥 − 1 − 3)(𝑥 + 1 − 3) = (𝑥 2 − (1 + 3))(𝑥 2 − (1 − 3)) = 𝑥 4 − 2𝑥 2 − 2

(c) Notice that 1 − 3 < 0 and ℚ(𝛼1 ) ⊂ ℝ and 𝛼2 is a complex number and therefore 𝛼2 ∉ ℚ(𝛼1 ) which implies
𝐾1 ≠ 𝐾2 . Since 𝐾1 ≠ 𝐾2 , then 𝐹 = 𝐾1 ∩ 𝐾2 is a proper subfield √of 𝐾1 and 𝐾√
2 which are both degree 4
extensions of ℚ, √
hence 𝐹 has degree 1 or 2, it is easy to see that 3 ∈ 𝐹 and 3 ∉ ℚ and hence we can
conclude 𝐹 = ℚ( 3)
[𝐾1 ∶ℚ]
(d) [𝐾1 ∶ 𝐹 ] = [𝐹 ∶ℚ] = 2, quadratic extensions are always Galois, similarly 𝐾2 is a Galois extension of 𝐹 ,
= 4
2
√ √ √ √ √
additionally this shows that 1 ± 3 are not squares in 𝐹 . Let 𝐾 = 𝐹 ( 1 + 3, 1 − 3) notice that 𝐾1 , 𝐾2
are proper subfields of 𝐾 , hence 𝐾1 𝐾2 ⊂ 𝐾 . Conversely, we know that [𝐾1 𝐾2 ∶ 𝐹 ] ≤ [𝐾1 ∶ 𝐹 ][𝐾2 ∶ 𝐹 ] = 4
therefore we must have 𝐾1 𝐾2 = 𝐾 . By the previous exercise we know that Gal(𝐾1 𝐾2 /𝐹 ) ≅ 𝐾4 . We can
explicitly write out the elements of Gal(𝐾1 𝐾2 /𝐹 ) as follows
{ { {
𝛼1 ↦ −𝛼1 𝛼1 ↦ 𝛼1 𝛼1 ↦ −𝛼1
𝐼 𝑑𝑒𝑛𝑡𝑖𝑡𝑦 𝜎1 = 𝜎2 = 𝜎3 = 𝜎1 𝜎2 = 𝜎2 𝜎1 =
𝛼2 ↦ 𝛼2 𝛼2 ↦ −𝛼2 𝛼2 ↦ −𝛼2

The subgroups are ⟨𝜎1 ⟩, ⟨𝜎2 ⟩, ⟨𝜎3 ⟩ with corresponding subfields 𝐹 (𝛼2 ), 𝐹 (𝛼1 ), 𝐹 (𝛼1 𝛼2 ) = 𝐹 ( −2)

(e) The splitting field of 𝑥 4 − 2𝑥 2 − 2 is 𝐾 = 𝐹 (𝛼1 , 𝛼2 ) and we know [𝐹 (𝛼1 , 𝛼2 ) ∶ 𝐹 ] = 4 and [𝐹 ∶ ℚ] = 2


from (c) and (d), therefore [𝐹 (𝛼1 , 𝛼2 ) ∶ ℚ] = [𝐹 (𝛼1 , 𝛼2 ) ∶ 𝐹 ][𝐹 ∶ ℚ] = 8. All that is left is to show that
Gal(𝐹 (𝛼1 , 𝛼2 )/ℚ) ≅ 𝐷8
In Chapter 14.6 we learn that Gal(𝐹 (𝛼1 , 𝛼2 )/ℚ) ↪ 𝑆4 , the reason being that a Galois extension permutes
the roots. Using this and the fact that 𝐷8 is the only subgroup of 𝑆4 with order 8, we conclude that
Gal(𝐹 (𝛼1 , 𝛼2 )/ℚ) ≅ 𝐷8

9
Question 2.17

Let 𝐾 /𝐹 be any finite extension and let 𝛼 ∈ 𝐾 . Let 𝐿 be a Galois extension of 𝐹 containing 𝐾 and let
𝐻 ≤ Gal(𝐿/𝐹 ) be the subgroup corresponding to 𝐾 . Define the norm of 𝛼 from 𝐾 to 𝐹 to be

𝑁𝐾 /𝐹 (𝛼) = ∏ 𝜎(𝛼)
𝜎

where the product is taken over all the embeddings of 𝐾 into an algebraic closure of 𝐹 (so over a set
of coset representatives for 𝐻 in Gal(𝐿/𝐹 ) by the Fundamental Theorem of Galois Theory). This is a
product of Galois conjugates of 𝛼. In particular, if 𝐾 /𝐹 is Galois this is ∏𝜎∈Gal(𝐾 /𝐹 ) 𝜎(𝛼)

(a) Prove that 𝑁𝐾 /𝐹 (𝛼) ∈ 𝐹


(b) Prove that 𝑁𝐾 /𝐹 (𝛼𝛽) = 𝑁𝐾 /𝐹 (𝛼)𝑁𝐾 /𝐹 (𝛽), so that the norm is a multiplicative map from 𝐾 to 𝐹
√ √
(c) Let 𝐾 = 𝐹 ( 𝐷) be a quadratic extension of 𝐹 . Show that 𝑁𝐾 /𝐹 (𝑎 + 𝑏 𝐷) = 𝑎2 − 𝐷𝑏2
(d) Let 𝑚𝛼 (𝑥) = 𝑥 𝑑 + ⋯ + 𝑎1 𝑥 + 𝑎0 ∈ 𝐹 [𝑥] be the minimal polynomial for 𝛼 ∈ 𝐾 over 𝐹 . Let 𝑛 = [𝐾 ∶ 𝐹 ].
Prove that 𝑑 divides 𝑛, that there are 𝑑 distinct Galois conjugates of 𝛼 which are all repeated 𝑛/𝑑
times in the product above and conclude that 𝑁𝐾 /𝐹 (𝛼) = (−1)𝑛 𝑎𝑛/𝑑
0

Solution.
(a) Let Ω = {𝜎 ∣ 𝜎𝐻 = 𝐻 , 𝐻 ≤ Gal(𝐿/𝐹 )} then 𝑁𝐾 /𝐹 (𝛼) = ∏𝜎∈Ω 𝜎(𝛼). Showing that 𝑁𝐾 /𝐹 (𝛼) ∈ 𝐹 is analogous
to showing that any 𝜏 ∈ Gal(𝐿/𝐹 ) fixes 𝑁𝐾 /𝐹 (𝛼) as 𝐹 is the fixed field of Gal(𝐿/𝐹 ).
Now, let 𝜏 ∈ Gal(𝐿/𝐹 ) we then have

𝜏(𝑁𝐾 /𝐹 (𝛼)) = 𝜏(∏ 𝜎(𝛼)) = ∏ 𝜏(𝜎(𝛼))


𝜎∈Ω 𝜎∈Ω

We can now notice that if 𝜏𝜎1 is in the same coset as 𝜏𝜎2 then 𝜏𝜎1 = 𝜏𝜎1 ℎ, ℎ ∈ 𝐻 which implies that 𝜎1 is
in the same coset as 𝜎2 , therefore {𝜎 ∣ 𝜎𝐻 = 𝐻 } = {𝜏𝜎 ∣ 𝜏𝜎𝐻 = 𝐻 } = Ω. Hence we can simplify

𝜏(𝑁𝐾 /𝐹 (𝛼)) = ∏ 𝜏(𝜎(𝛼)) = ∏ 𝜎(𝛼) = 𝑁𝐾 /𝐹 (𝛼)


𝜎∈Ω 𝜎∈Ω

We have now shown that 𝑁𝐾 /𝐹 (𝛼) is fixed by arbitrary 𝜏 ∈ Gal(𝐿/𝐹 )


(b) 𝜎 is a homomorphism (remember that an embedding is just an injective homomorphism) and therefore

𝑁𝐾 /𝐹 (𝛼𝛽) = ∏ 𝜎(𝛼𝛽) = ∏ 𝜎(𝛼)𝜎(𝛽) = 𝑁𝐾 /𝐹 (𝛼)𝑁𝐾 /𝐹 (𝛽)


𝜎 𝜎


(c) If 𝐾 = 𝐹 ( 𝐷) is a quadratic
√ extension
√ then we have 2 embeddings. Namely, 𝜎, 𝜏 where 𝜎 is identity and
𝜏 which fixes 𝐹 and maps 𝐷 ↦ − 𝐷, hence
√ √ √ √ √
𝑁𝐾 /𝐹 (𝑎 + 𝐷) = 𝜎(𝑎 + 𝑏 𝐷)𝜏(𝑎 + 𝑏 𝐷) = (𝑎 + 𝑏 𝐷)(𝑎 − 𝑏 𝐷) = 𝑎2 − 𝑏2 𝐷

(d) 𝑚𝛼 (𝑥) has degree 𝑑 and therefore [𝐹 (𝛼) ∶ 𝐹 ] = 𝑑 which divides [𝐾 ∶ 𝐹 ] = 𝑛. Let 𝑚𝛼 (𝑥) = 𝑥 𝑑 + 𝑎𝑑−1 𝑥 𝑑−1 +
⋯ + 𝑎0 , where 𝑎𝑖 ∈ 𝐹 . Consider 𝐻 = {𝜎 ∈ 𝐺 ∣ 𝜎(𝛼) = 𝛼} and notice that it is a subgroup of 𝐺. For any
𝜎 ∈ 𝐺, it must be that 𝜎 ∶ 𝛼 ↦ 𝛼𝑖 , where 𝛼1 = 𝛼, … , 𝛼𝑑 are the roots of 𝑚𝛼 (𝑥). Since 𝐾 /𝐹 is Galois, then
any irreducible polynomial over 𝐹 is separable, and thus we can conclude that the 𝛼𝑖 ’s are distinct.
Now consider 𝐺 acting on 𝐾 in the obvious way (That is 𝜎 ⋅ 𝛼 = 𝜎(𝛼)). Then notice that 𝐻 = Stab(𝛼), and
by orbit-stabiliser theorem, we have

|𝐺| = |𝐻 ||(𝛼)|
𝑛 = |𝐻 |(𝑑) there are 𝑑 distinct roots
𝑛
|𝐻 | =
𝑑

10
Then
𝑁𝐾 /𝐹 (𝛼) = ∏ 𝜎(𝛼)
𝜎∈𝐺
𝑑
= ∏ ∏(𝜎𝑖 𝜏)(𝛼)
𝑖=1 𝜏∈𝐻
𝑑
𝑛
= ∏ (𝜎𝑖 (𝛼)) 𝑑 𝜏(𝛼) = 𝛼, ∀𝜏 ∈ 𝐻
𝑖=1
𝑑 𝑛
= ∏ 𝛼𝑖𝑑
𝑖=1
𝑛 𝑛 𝑛
Since 𝑎0 = (−1)𝑑 ∏𝑑𝑖=1 𝛼𝑖 , then it follows that 𝑁𝐾 /𝐹 (𝛼) = (∏𝑑𝑖=1 𝛼𝑖 ) 𝑑 = ((−1)𝑑 𝑎0 ) 𝑑 = (−1)𝑛 𝑎0𝑑 as desired.

3 Chapter 14.3

Question 3.1

Factor 𝑥 8 − 𝑥 into irreducibles in ℤ[𝑥] and 𝔽2 [𝑥]

Solution. In ℤ[𝑥] we have 𝑥 8 − 𝑥 = 𝑥(𝑥 7 − 1) = 𝑥 ⋅ Φ1 (𝑥) ⋅ Φ7 (𝑥) = 𝑥(𝑥 − 1)(𝑥 6 + 𝑥 5 + 𝑥 4 + 𝑥 3 + 𝑥 2 + 𝑥 + 1).


From the discussion of Proposition 18 we have 𝑥 8 − 𝑥 = 𝑥(𝑥 − 1)(𝑥 3 + 𝑥 + 1)(𝑥 3 + 𝑥 2 + 1) in 𝔽2 [𝑥] □

Question 3.2

Write out the multiplication table for 𝔽4 and 𝔽8

Solution. We know 𝑥 4 − 𝑥 = 𝑥(𝑥 − 1)(𝑥 2 + 𝑥 + 1) and 𝑔(𝑥) = 𝑥 2 + 𝑥 + 1 is irreducible in 𝔽2 [𝑥]. Let 𝜃 be a root
of 𝑔(𝑥), we then have
𝔽4 ≅ 𝔽2 [𝑥]/(𝑥 2 + 𝑥 + 1) ≅ 𝔽2 (𝜃) = {𝑎 + 𝑏𝜃 ∣ 𝑎, 𝑏 ∈ 𝔽2 } = {0, 1, 𝜃, 1 + 𝜃}
Using 𝜃2 + 𝜃 + 1 = 0 we then have the multiplication table:
× 0 1 𝜃 𝜃+1
0 0 0 0 0
1 0 1 𝜃 𝜃+1
𝜃 0 𝜃 𝜃+1 1
𝜃+1 0 𝜃+1 1 𝜃
From Question 3.1 we have 𝑥 8 − 𝑥 = 𝑥(𝑥 − 1)(𝑥 3 + 𝑥 + 1)(𝑥 3 + 𝑥 2 + 1) and ℎ(𝑥) = 𝑥 3 + 𝑥 + 1 is irreducible in
𝔽2 [𝑥], let 𝛼 be a root of ℎ(𝑥). We then have
𝔽8 ≅ 𝔽2 (𝛼) ≅ 𝔽2 [𝑥]/(𝑥 3 + 𝑥 + 1) ≅ {𝑎 + 𝑏𝛼 + 𝑐𝛼 2 ∣ 𝑎, 𝑏, 𝑐 ∈ 𝔽2 } = {0, 1, 𝛼, 𝛼 + 1, 𝛼 2 , 𝛼 2 + 1, 𝛼 2 + 𝛼, 𝛼 2 + 𝛼 + 1}
Using 𝛼 3 + 𝛼 + 1 = 0, we have the multiplication table:
× 0 1 𝛼 𝛼+1 𝛼2 𝛼2 + 1 𝛼2 + 𝛼 𝛼2 + 𝛼 + 1
0 0 0 0 0 0 0 0 0
1 0 1 𝛼 𝛼+1 𝛼2 𝛼2 + 1 𝛼2 + 𝛼 𝛼2 + 𝛼 + 1
𝛼 0 𝛼 𝛼2 𝛼2 + 𝛼 𝛼+1 1 𝛼2 + 𝛼 + 1 𝛼2 + 1
2
𝛼+1 0 𝛼+1 𝛼 +𝛼 𝛼2 + 1 𝛼2 + 𝛼 + 1 𝛼2 1 𝛼
𝛼2 0 𝛼2 𝛼+1 2
𝛼 +𝛼+1 𝛼2 + 𝛼 𝛼 𝛼2 + 1 1
2 2
𝛼 +1 0 𝛼 +1 1 𝛼2 𝛼 𝛼2 + 𝛼 + 1 𝛼+1 𝛼2 + 𝛼
𝛼2 + 𝛼 0 𝛼2 + 𝛼 𝛼2 + 𝛼 + 1 1 𝛼2 + 1 𝛼+1 𝛼 𝛼2
𝛼2 + 𝛼 + 1 0 𝛼2 + 𝛼 + 1 𝛼2 + 1 𝛼 1 𝛼2 + 𝛼 𝛼2 𝛼+1

11

Question 3.3

Prove that an algebraically closed field must be infinite

Solution.
𝑛
(Method 1) Suppose 𝐾 is a finite algebraically closed field, then 𝐾 ≅ 𝔽𝑝𝑛 = {𝛼 ∣ 𝛼 𝑝 − 𝛼 = 0}. Let 𝛼0 , 𝛼1 ⋯ 𝛼𝑛
be the distinct roots and hence all the elements of 𝐾 , then 𝑓 (𝑥) = 1 + ∏𝑛𝑖=0 (𝑥 − 𝛼𝑖 ) has no root in
𝐾 [𝑥] which contradicts the assumption that 𝐾 is algebraically closed.
𝑚
(Method 2) Alternatively, for a field to be algebraically closed, it necessarily must contain roots of 𝑥 𝑝 − 𝑥 for
𝑚
any 𝑚 and for any prime 𝑝. Since each 𝑥 𝑝 − 𝑥 has 𝑝𝑚 distinct roots, then |𝔽| ≥ 𝑝𝑚 for any 𝑝, 𝑚.
That is, it must be infinite.
(Method 3) Alternatively, we proceed by contraposition. Fix some arbitrary finite field 𝔽𝑝𝑛 . Let 𝑞 be a prime
s.t. 𝑞 ∤ 𝑛. By proposition 17, ∃𝑄(𝑥) ∈ 𝔽𝑝 irreducible and of degree 𝑞. Fix any 𝛼 ∈ 𝔽𝑝𝑛 . If 𝑄(𝛼) = 0,
then we have the following.
𝔽𝑝 ⊆ 𝔽𝑝 (𝛼) ⊆ 𝔽𝑝𝑛
where the degree of the first extension is 𝑞. But 𝑞 ∤ 𝑛 and thus cannot be the case.

Question 3.4

Construct the finite field of 16 elements and find a generator for the multiplicative group. How many
generators are there?

Solution. A finite field with 16 elements will be isomorphic to 𝔽24 . Again by the discussion of Proposition 18
we have 𝑥 16 − 𝑥 = 𝑥(𝑥 − 1)(𝑥 2 + 𝑥 + 1)(𝑥 4 + 𝑥 3 + 1)(𝑥 4 + 𝑥 + 1)(𝑥 4 + 𝑥 3 + 𝑥 2 + 𝑥 + 1) and 𝑓 (𝑥) = 𝑥 4 + 𝑥 + 1 is
irreducible in 𝔽2 [𝑥], let 𝜃 be a root of 𝑓 (𝑥), hence we have

𝔽16 ≅ 𝔽2 [𝑥]/(𝑓 (𝑥)) ≅ 𝔽2 (𝜃) =


{0, 1, 𝜃, 𝜃2 , 𝜃3 , 1 + 𝜃, 1 + 𝜃2 , 1 + 𝜃3 , 𝜃 + 𝜃2 , 𝜃 + 𝜃3 , 𝜃2 + 𝜃3 , 1 + 𝜃 + 𝜃2 , 1 + 𝜃 + 𝜃3 , 1 + 𝜃2 + 𝜃3 , 𝜃 + 𝜃2 + 𝜃3 , 1 + 𝜃 + 𝜃2 + 𝜃3 }

Now we can notice that 𝑥 3 ≠ 𝑥, 𝑥 5 = 𝑥 + 𝑥 2 ≠ 𝑥 hence ord(𝑥) ≠ 3 or 5 but it must divide 15 = |𝔽×16 |,
hence ord(𝑥) = 15 and ⟨𝑥⟩ generates 𝔽×16 , therefore we conclude ⟨𝑥⟩ ≅ ℤ15 and hence there will be 𝜑(15) = 8
generators, they are {𝑥 𝑎 ∣ gcd(𝑎, 15) = 1} = {𝑥 1 , 𝑥 2 , 𝑥 4 , 𝑥 7 , 𝑥 8 , 𝑥 11 , 𝑥 13 , 𝑥 14 } □

Question 3.5

Exhibit an explicit isomorphism between the splitting fields of 𝑥 3 − 𝑥 + 1 and 𝑥 3 − 𝑥 − 1 over 𝔽3

Solution. Notice that 𝑓 (𝑥) = 𝑥 3 − 𝑥 + 1 and 𝑔(𝑥) = 𝑥 3 − 𝑥 − 1 are both irreducible in 𝔽3 [𝑥] because
𝑓 (0) = 𝑓 (1) = 𝑓 (2) = 1 ≠ 0 and 𝑔(0) = 𝑔(1) = 𝑔(2) = −1 = 2 ≠ 0, therefore we have

𝔽27 ≅ 𝔽3 [𝑥]/(𝑓 (𝑥)) ≅ 𝔽3 [𝑥]/(𝑔(𝑥))

Let 𝛼(𝑥) = 𝑎𝑥 2 + 𝑏𝑥 + 𝑐 be a root of 𝑓 (𝑥) in 𝔽3 [𝑥]/(𝑔(𝑥)), then if we map 𝑥 ∈ 𝔽3 /(𝑓 (𝑥)) ↦ 𝛼(𝑥) we have our

12
isomorphism. Now, we need to find 𝛼(𝑥) such that 𝑓 (𝛼(𝑥)) = 0

𝑓 (𝛼(𝑥)) = (𝑎𝑥 2 + 𝑏𝑥 + 𝑐)3 − (𝑎𝑥 2 + 𝑏𝑥 + 𝑐) + 1


= 𝑎𝑥 6 + 𝑏𝑥 3 + 𝑐 − 𝑎𝑥 2 − 𝑏𝑥 − 𝑐 + 1 (𝑑 3 = 𝑑 for 𝑑 ∈ 𝔽3 )
= 𝑎(𝑥 + 1)2 + 𝑏(𝑥 + 1) + 𝑐 − 𝑎𝑥 2 − 𝑏𝑥 − 𝑐 + 1 (𝑥 3 = 𝑥 + 1 in 𝔽3 /(𝑔(𝑥))
= 𝑎𝑥 2 + 2𝑎𝑥 + 𝑎 + 𝑏𝑥 + 𝑏 + 𝑐 − 𝑎𝑥 2 − 𝑏𝑥 − 𝑐 + 1
= 2𝑎𝑥 + 𝑏 + 𝑎 + 1 = 0

Therefore we have 𝑎 = 0, 𝑏 = 2, then we just let 𝑐 = 0 and we have 𝛼(𝑥) = 2𝑥, we finally have the explicit
isomorphism

𝔽3 [𝑥]/(𝑓 (𝑥)) ↦ 𝔽3 [𝑥]/(𝑔(𝑥))


𝑥 ↦ 2𝑥

Question 3.6 - Unfinished

√ √ √
Suppose
√ 𝐾√ = ℚ(𝜃) = ℚ( 𝐷1 , 𝐷2 ) with 𝐷1 , 𝐷2 ∈ ℤ, is a biquadratic extension and that 𝜃 = 𝑎 + 𝑏 𝐷1 +
𝑐 𝐷2 + 𝑑 𝐷1 𝐷2 where 𝑎, 𝑏, 𝑐, 𝑑 ∈ ℤ are integers. Prove that the minimal polynomial 𝑚𝜃 (𝑥) for 𝜃 over
ℚ is irreducible of degree 4 over ℚ but is reducible modulo every prime 𝑝. In particular show that the
polynomial 𝑥 4 − 10𝑥 2 + 1 is irreducible in ℤ[𝑥] but is reducible modulo every prime. [Use the fact that
there are no biquadratic extensions over finite fields.]

Question 3.7

Prove that one of 2, 3 or 6 is a square in 𝔽𝑝 for every prime 𝑝. Conclude that the polynomial

𝑥 6 − 11𝑥 4 + 36𝑥 2 − 36 = (𝑥 2 − 2)(𝑥 2 − 3)(𝑥 2 − 6)

has a root modulo 𝑝 for every prime 𝑝 but has no root in ℤ

Solution. Let ⟨𝑥⟩ = 𝔽×𝑝 . Then 𝑎 ∈ 𝔽×𝑝 is a square if and only if 𝑎 = 𝑥 2𝑘 = (𝑥 𝑘 )2 where 𝑏 ∈ 𝔽×𝑝 , 𝑘 ∈ ℤ. Now
suppose 2 and 3 are not square in 𝔽𝑝 , this means 2 = 𝑥 2𝑘1 +1 and 3 = 𝑥 2𝑘2 +1 where 𝑘1 , 𝑘2 ∈ ℤ and therefore
6 = 𝑥 2(𝑘1 +𝑘2 +1) is a square in 𝔽𝑝 .
Therefore, 2, 3 or 6 must be a square in 𝔽𝑝 and hence WLOG suppose 2 is a square in 𝔽𝑝 there is an 𝑎 ∈ 𝔽𝑝
such that 𝑎2 − 2 =√0 which √ implies
√ 𝑎 is a root of (𝑥 2 − 2)(𝑥 2 − 3)(𝑥 2 − 6). Furthermore, the real roots of this
polynomial are ± 2, ± 3, ± 6 which are all not integers, hence the polynomial has no root in ℤ. □

13

You might also like